آيا شما مايل به راه افتادن تاپيك ماراتن نامساوي مي باشي


  • مجموع رای دهندگان
    339

Dadgarnia

New Member
ارسال ها
1,350
لایک ها
1,127
امتیاز
0
پاسخ : ماراتن نامساوي

سوال بعد:
اگر
اعدادي مثبت با شرط
باشند نشان دهيد:

 

aras2213

New Member
ارسال ها
216
لایک ها
228
امتیاز
0
پاسخ : ماراتن نامساوي

سوال بعد:
اگر
اعدادي مثبت با شرط
باشند نشان دهيد:

هی میخوام فیزیک بخونم نمیشه:33::4:
ابتدا دقت کنید که:
.پس کافی است ثابت کنیم که:

حالا طبق کوشی داریم:



و
.از جمع این دو نامساوی حکم نتیجه میشه.

---- دو نوشته به هم متصل شده است ----

سوال بعد:
 

math1998

New Member
ارسال ها
336
لایک ها
224
امتیاز
0
پاسخ : ماراتن نامساوي

هی میخوام فیزیک بخونم نمیشه:33::4:
ابتدا دقت کنید که:
.پس کافی است ثابت کنیم که:

حالا طبق کوشی داریم:



و
.از جمع این دو نامساوی حکم نتیجه میشه.

---- دو نوشته به هم متصل شده است ----

سوال بعد:


حالا میتونیم نامساوی رو خطی کنیم و با شرط مجموع یک همگنش کنیم چرا که تو هر مرحله میتونیم با استفاده از این درجه های دو طرف رو برابر کنیم و بعد هم مورهد میزنیم!!!!
 

aras2213

New Member
ارسال ها
216
لایک ها
228
امتیاز
0
پاسخ : ماراتن نامساوي



حالا میتونیم نامساوی رو خطی کنیم و با شرط مجموع یک همگنش کنیم چرا که تو هر مرحله میتونیم با استفاده از این درجه های دو طرف رو برابر کنیم و بعد هم مورهد میزنیم!!!!
ببخشید که من نمیفهمم یعنی شما میگید که
؟ اگه این طوریه این نامساوی برای a=b=c=1/3 نادرسته.
 

math1998

New Member
ارسال ها
336
لایک ها
224
امتیاز
0
پاسخ : ماراتن نامساوي

ببخشید که من نمیفهمم یعنی شما میگید که
؟ اگه این طوریه این نامساوی برای a=b=c=1/3 نادرسته.
حق با شماست اخه کامل حل نکردم نامساوی رو یکم ضعیف کردم با اون نامساوی حسابی-هندسی اولی اما مهم نیست با همون خطی کردن و همگن کردن سوال اصلی و سپس مورهد زدن هم حله!!!
 

Dadgarnia

New Member
ارسال ها
1,350
لایک ها
1,127
امتیاز
0
پاسخ : ماراتن نامساوي

حق با شماست اخه کامل حل نکردم نامساوی رو یکم ضعیف کردم با اون نامساوی حسابی-هندسی اولی اما مهم نیست با همون خطی کردن و همگن کردن سوال اصلی و سپس مورهد زدن هم حله!!!
ببخشيد ميشه راهتون رو كامل بنويسين؟ آخه من الان اينو خطيش كردم ولي نتونستم با مورهد كار خاصي انجام بدم.
 

aras2213

New Member
ارسال ها
216
لایک ها
228
امتیاز
0
پاسخ : ماراتن نامساوي

ببخشيد ميشه راهتون رو كامل بنويسين؟ آخه من الان اينو خطيش كردم ولي نتونستم با مورهد كار خاصي انجام بدم.
به نظر من دقیقا نکته همین جاست!به یه چیز قوی تر از مورهد نیاز داریم!(البته من با مورهد امتحان نکردم ولی حلی که ازش میدونم با یه چیز(که نمیگم چیه تا فکر کنید:205:) قوی تر از مورهده)
 

Dadgarnia

New Member
ارسال ها
1,350
لایک ها
1,127
امتیاز
0
پاسخ : ماراتن نامساوي

چون كسي راه حلي نذاشت من راه حل زشت خودمو مي ذارم.
قرار مي ديم
حالا با خطي كردن نامساوي داريم:
اين يه چند جمله اي درجه دو بر حسب q هست پس براي نا منفي بودن كافيه كه q كوچكتر از ريشه هاي اون باشه. از طرف ديگه مي دونيم:
پس كافيه ثابت كنيم:

با استفاده از شرط سوال مي دونيم كه
هست پس طرف چپ مثبته و مي تونيم دو طرف رو به توان دو برسونيم حالا با ساده كردن عبارت بدست اومده داريم:

با توجه به شرط سوال واضحه كه هر دو عبارت بدست اومده نامنفي هستند پس نامساوي ثابت ميشه.

---- دو نوشته به هم متصل شده است ----

سوال بعد:
براي اعداد مثبت
با شرط
ثابت كنيد:

 

aras2213

New Member
ارسال ها
216
لایک ها
228
امتیاز
0
پاسخ : ماراتن نامساوي

با توجه به شرط سوال و نامساوی حسابی-هندسی،داریم:


---- دو نوشته به هم متصل شده است ----

با فرض
،بیشترین مقدار
را بدست آورید.

a,b,c نامنفی ان.
 
آخرین ویرایش توسط مدیر

m-saghaei

New Member
ارسال ها
338
لایک ها
258
امتیاز
0
پاسخ : ماراتن نامساوي

با فرض
،بیشترین مقدار
را بدست آورید.

a,b,c نامنفی ان.
داریم

حالا میایم اینو یه جوری به حکم سوال نزدیک میکنیم! اینجوری:



به طرفین 18 اضافه میکنیم.پس میشه:



پس
که اگه سه تاشو با هم جمع کنیم مینیمم که
ه به دست میاد!

---- دو نوشته به هم متصل شده است ----

سوال بعد:

اعداد حقیقی تو بازه ی
هستن به طوریکه
. ثابت کنید:

 
آخرین ویرایش توسط مدیر

REZA 73

Active Member
ارسال ها
139
لایک ها
184
امتیاز
43
پاسخ : ماراتن نامساوي

داریم

حالا میایم اینو یه جوری به حکم سوال نزدیک میکنیم! اینجوری:



به طرفین 18 اضافه میکنیم.پس میشه:



پس
که اگه سه تاشو با هم جمع کنیم مینیمم که
ه به دست میاد!

---- دو نوشته به هم متصل شده است ----

سوال بعد:

اعداد حقیقی تو بازه ی
هستن به طوریکه
. ثابت کنید:

اگه سمت چپ نامساوی را A بنامیم با یه کوشی داریم:
پس کافیست ثابت کنیم:
پس از کمی ساده کردن باید ثابت کنیم:
که با استفاده از نامساوی های زیر این رابطه بدیهی است:



 

m-saghaei

New Member
ارسال ها
338
لایک ها
258
امتیاز
0
پاسخ : ماراتن نامساوي

سوال بعد:
اگر
طول اضلاع يك مثلث باشند، ثابت كنيد:

برای بزرگتر از دو اش:

میدونیم پای (a+b-c) مثبته

حالا اگه بازش کنیم و به
تقسیمش کنیم میشه:

پس اگه بگیم نامساوی
برقراره مسئله حله.

سادش کنیم درمیاد
که اینم به وضوح برقراره!

واسه کوچکتر مساوی 3 هم فکر کنم باید اینکارو بکنیم:


که تو صورت اویلر بشه و به اون سه تا کسر قبلیش هم یه یک اضافه کنیم.بعدش
رو فاکتور بگیریم و ...

ولی یه جاییش گیر کردم!
 
آخرین ویرایش توسط مدیر

REZA 73

Active Member
ارسال ها
139
لایک ها
184
امتیاز
43
پاسخ : ماراتن نامساوي

دوستمون نامساوی سمت چپ رو ثابت کرد حالا برای سمت راست:


حالا با جایگذاری باید ثابت کنیم که :
پس از ساده سازی به این میرسیم که:
دوباره پس از ساده کردن داریم:
که این هم بدیهیه.
 

m-saghaei

New Member
ارسال ها
338
لایک ها
258
امتیاز
0

Dadgarnia

New Member
ارسال ها
1,350
لایک ها
1,127
امتیاز
0
پاسخ : ماراتن نامساوي

پس دوستان لطفا به فكر كردن روي طرف چپ نامساوي ادامه بدين (البته فكر كنم بشه راه آقاي سقايي رو كامل كرد.)!
 

REZA 73

Active Member
ارسال ها
139
لایک ها
184
امتیاز
43
پاسخ : ماراتن نامساوي

با همون تغییر متغیر ها اون سمت هم ثابت میشه:
در حقیقت باید ثابت بشه:

با ساده سازی داریم:
که باز هم با ساده کردن به رابطه بدیهی زیر میرسیم:

 
آخرین ویرایش توسط مدیر

Dadgarnia

New Member
ارسال ها
1,350
لایک ها
1,127
امتیاز
0
پاسخ : ماراتن نامساوي

با همون تغییر متغیر ها اون سمت هم ثابت میشه:
در حقیقت باید ثابت بشه:

با ساده سازی داریم:
که باز هم با ساده کردن به رابطه بدیهی زیر میرسیم:

بله درسته. براي ديدن راه حل هاي بيشتر مي تونين به اين لينك مراجعه كنيد: AoPS Forum - Nice (triangle inequality) • Art of Problem Solving
سوال بعد:
حداقل مقدار
را بيابيد به طوريكه براي هر چهار عدد مثبت
و با شرط
نامساوي زير برقرار باشد:

 
بالا